织梦行云

  博客园  :: 首页  :: 新随笔  :: 联系 :: 订阅 订阅  :: 管理

In mathematics, in the areas of order theory and combinatorics, Dilworth's theorem characterizes the width of any finite partially ordered set in terms of a partition of the order into a minimum number of chains. It is named for the mathematician Robert P. Dilworth.

An antichain in a partially ordered set is a set of elements no two of which are comparable to each other, and a chain is a set of elements every two of which are comparable. Dilworth's theorem states that there exists an antichain A, and a partition of the order into a family P of chains, such that the number of chains in the partition equals the cardinality of A. When this occurs, A must be the largest antichain in the order, for any antichain can have at most one element from each member of P.

Similarly, P must be the smallest family of chains into which the order can be partitioned, for any partition into chains must have at least one chain per element of A. The width of the partial order is defined as the common size of A and P.

An equivalent way of stating Dilworth's theorem is that, in any partially ordered set, the maximum number of elements in any antichain equals the minimum number of chains in any partition of the set into chains.

The following proof by induction on the size of P is based on that of Galvin (1994).

Let P be a finite partially ordered set. The theorem holds trivially if P is empty. So, assume that P has at least one element, and let a be a maximal element of P.

By induction, we assume that for some integer k the partially ordered set P':=P\setminus\{a\} can be covered by k disjoint chains C_1,\dots,C_k and has at least one antichain A0 of size k.

Clearly, A_0\cap C_i\ne\emptyset for i=1,2,\dots,k. For i=1,2,\dots,k, let xi be the maximal element in Ci that belongs to an antichain of length k in P', and set . We claim that A is an antichain. Let A1 be an antichain of length k that contains x1. Then A_1\cap C_2\ne\emptyset. Let y\in A_1\cap C_2. Then y\le x_2, by the definition of x2. This implies that x_1\not \ge x_2, since x_1\not\ge y. Similarly, we have x_i\not\ge x_j for all i\ne j in \{1,2,\dots,k\}, verifying that A is an antichain.

We now return to P. Suppose first that a\ge x_i for some i\in\{1,2,\dots,k\}. Let K be the chain \{a\}\cup\{z\in C_i:z\le x_i\}. Then by the choice of xi, P\setminus K does not have an antichain of length k. Induction then implies that P\setminus K can be covered by k − 1 disjoint chains. Thus, P can be covered by k disjoint chains, as required. Next, if a\not\ge x_i for each i\in\{1,2,\dots,k\}, then A\cup\{a\} is an antichain of length k + 1 in P (since a is maximal in P). Because P can be covered by the k + 1 chains \{a\},C_1,C_2,\dots,C_k, this completes the proof.

posted on 2010-03-16 14:33  BloodElf  阅读(234)  评论(0编辑  收藏  举报